Boundary Conditions for Wave Functions in Infinite Potential Wells

In summary, the wave function in a potential that is infinite at x \leq 0 must be zero at x = 0 and x = \infty. However, a wave function that goes to -\infty at x = -\infty will not fit in the (time independent) Schrödinger equation, whereas one that goes to zero at -\infty does. The potential is the harmonic oscillator on the positive semiaxis, and the problem is what are the mathematical requirements for the wave function. Let's say you have a function \psi(x), then what are the mathematical requirements that function need to meet in order to be a wavefunction for that potential? Physical states are described
  • #1
Spinny
20
0
Hi, I have a question about the mathematical requirements of a wave function in a potential that is infinite at [tex]x \leq 0[/tex]. (At the other side it goes towards infinity at [tex]x = \infty[/tex].) Now, given a wave function in this potential that is zero for [tex]x = 0[/tex] and [tex]x = \infty[/tex]. Does it matter what that wavefunction is at [tex]x = -\infty[/tex]? I mean, I just figured you would have a wave function there that's zero all the way. Why will a wave function that goes to [tex]-\infty[/tex] at [tex]x = -\infty[/tex] not fit in the (time independent) Schrödinger equation, whereas one that goes to zero at [tex]-\infty[/tex] does? After all when we're normalizing it, we're just integrating from 0 to [tex]\infty[/tex] and doesn't really need to bother with it at negative x values. Or is that just some mathematical requirement that is independent of the physical properties? Can someone enlighten me, please?
 
Physics news on Phys.org
  • #2
Spinny said:
Hi, I have a question about the mathematical requirements of a wave function in a potential that is infinite at [tex]x \leq 0[/tex]. (At the other side it goes towards infinity at [tex]x = \infty[/tex].) Now, given a wave function in this potential that is zero for [tex]x = 0[/tex] and [tex]x = \infty[/tex]. Does it matter what that wavefunction is at [tex]x = -\infty[/tex]? I mean, I just figured you would have a wave function there that's zero all the way. Why will a wave function that goes to [tex]-\infty[/tex] at [tex]x = -\infty[/tex] not fit in the (time independent) Schrödinger equation, whereas one that goes to zero at [tex]-\infty[/tex] does? After all when we're normalizing it, we're just integrating from 0 to [tex]\infty[/tex] and doesn't really need to bother with it at negative x values. Or is that just some mathematical requirement that is independent of the physical properties? Can someone enlighten me, please?

Is this your problem:
"Solve the unidimensional SE for one particle in the the potential field:

[tex] U(x)=\left\{\begin{array}{c}+\infty,\mbox{for} \ x\in(-\infty,0]\\0,\mbox{for} \ x\in (0,+\infty)\end{array}\right [/tex]

,because you didn't say anything about the potential in the positive semiaxis...

Daniel.
 
  • #3
dextercioby said:
Is this your problem:
"Solve the unidimensional SE for one particle in the the potential field:

[tex] U(x)=\left\{\begin{array}{c}+\infty,\mbox{for} \ x\in(-\infty,0]\\0,\mbox{for} \ x\in (0,+\infty)\end{array}\right [/tex]

,because you didn't say anything about the potential in the positive semiaxis...

Daniel.

The potential is the harmonic oscillator on the positive semiaxis. The problem is what are the mathematical requirements for the wave function. Let's say you have a function [tex]\psi(x)[/tex], then what are the mathematical requirements that function need to meet in order to be a wavefunction for that potential?
 
  • #4
Physical states are described by normalizable wavefunctions...

In your case,on the negative semiaxis the wave function is zero and on the positive semiaxis is a Hermite polynomial.So i'd say this is normalizable.

Then comes the continuity of the wavefunction.Both 0 & Hermite Polynomials are continuous,however,at the point 0,the continuity must be enforced.

The first derivative issue is rather tricky.U may want to consult a book how to deal with infinite potentials & the conditions imposed on the wavefunction.

Daniel.
 
  • #5
Think about the following:

1. The eigenfunctions for the linear oscillator are strictly even or or odd.

2. For this problem, why should there be any boundary condition on the momentum, the first spatial derivative, at x=0, if two boundary conditions have already been imposed? (Think about a particle wave packet, in the oscillator well, moving toward the x=0 wall. What's going to happen at the wall?)

Regards,
Reilly Atkinson
 

1. What is a wave function?

A wave function is a mathematical representation of a particle or system in quantum mechanics. It describes the probability of finding a particle in a particular state or location.

2. What are the requirements for a valid wave function?

A valid wave function must be continuous, single-valued, and square-integrable. It must also satisfy the normalization condition, which means that the integral of the square of the wave function over all space must equal 1.

3. Can a wave function have a negative value?

Yes, a wave function can have negative values. However, the probability of finding a particle at a specific point is proportional to the square of the wave function, so the overall probability is always positive.

4. What is the significance of the wave function in quantum mechanics?

The wave function is a fundamental concept in quantum mechanics as it allows us to calculate the probability of finding a particle in a particular state or location. It also plays a crucial role in determining the behavior of particles on a quantum level.

5. How is the wave function related to the uncertainty principle?

The wave function is related to the uncertainty principle through the concept of superposition. According to the uncertainty principle, it is impossible to know the exact position and momentum of a particle simultaneously. The wave function describes the probability of finding a particle in a particular state, but it does not provide information about the particle's specific position or momentum at any given moment.

Similar threads

Replies
24
Views
530
Replies
7
Views
863
  • Quantum Physics
Replies
3
Views
928
  • Quantum Physics
Replies
10
Views
1K
Replies
8
Views
1K
Replies
10
Views
1K
Replies
4
Views
877
  • Quantum Physics
Replies
31
Views
3K
Replies
18
Views
7K
  • Quantum Physics
Replies
1
Views
747
Back
Top